LSAT and Law School Admissions Forum

Get expert LSAT preparation and law school admissions advice from PowerScore Test Preparation.

User avatar
 Dave Killoran
PowerScore Staff
  • PowerScore Staff
  • Posts: 5853
  • Joined: Mar 25, 2011
|
#43086
Complete Question Explanation
(The complete setup for this game can be found here: lsat/viewtopic.php?f=1250&t=16333)

The correct answer choice is (B).

In this question, the four incorrect answers all could occur, and the one correct answer cannot occur.


Answer choice (B): This is the correct answer choice. As discussed during the setup, when J and M are both selected, they must be a block (per the fourth rule), and thus they have only one possible alignment—M in second, J in third:

  • With J in the group, J must be third from the third rule. M must then be second or fourth, on first glance. But, from the first rule, H and T cannot be consecutive, and so they cannot jointly occupy the first two spaces. Thus, one of H and T must be earlier than J, and the other must later, which is fourth. This forces M into second.


Consequently, there's never a scenario where M is visited at some time after J, and thus answer choice (B) cannot occur and is correct.

Get the most out of your LSAT Prep Plus subscription.

Analyze and track your performance with our Testing and Analytics Package.